Guest

is there any proof or derivation of L hospital rule

is there any proof or derivation of L hospital rule

Grade:

1 Answers

SAGAR SINGH - IIT DELHI
878 Points
13 years ago

Dear vikash,

Let lim stand for the limit  lim_(x->c), lim_(x->c^-), lim_(x->c^+), lim_(x->infty), or lim_(x->-infty), and suppose that lim f(x) and lim g(x) are both zero or are both +/-infty. If

 lim(f^'(x))/(g^'(x))

has a finite value or if the limit is +/-infty, then

 lim(f(x))/(g(x))=lim(f^'(x))/(g^'(x)).

Historically, this result first appeared in l'Hospital's 1696 treatise, which was the first textbook on differential calculas.

 

We are all IITians and here to help you in your IIT JEE preparation.

All the best.

 If you like this answer please approve it....

win exciting gifts by answering the questions on Discussion Forum

 

Sagar Singh

B.Tech IIT Delhi

Think You Can Provide A Better Answer ?

ASK QUESTION

Get your questions answered by the expert for free